K
Khách

Hãy nhập câu hỏi của bạn vào đây, nếu là tài khoản VIP, bạn sẽ được ưu tiên trả lời.

AH
Akai Haruma
Giáo viên
27 tháng 9 2023

Lời giải:

Từ $\frac{1}{x}+\frac{1}{y}+\frac{1}{z}=0$

$\Rightarrow xy+yz+xz=0$

Khi đó:

$x^2+2yz=x^2+yz-xz-xy=(x^2-xy)-(xz-yz)=x(x-y)-z(x-y)=(x-z)(x-y)$

Tương tự với $y^2+2zx, z^2+2xy$ thì:

$P=\frac{yz}{(x-z)(x-y)}+\frac{xz}{(y-z)(y-x)}+\frac{xy}{(z-x)(z-y)}$

$=\frac{-yz(y-z)-xz(z-x)-xy(x-y)}{(x-y)(y-z)(z-x)}=\frac{-[yz(y-z)+xz(z-x)+xy(x-y)]}{-[xy(x-y)+yz(y-z)+xz(z-x)]}=1$

18 tháng 1 2016

đề bài sai, phải là 1/x+1/y+1/z=1/3 chứ

18 tháng 1 2016

em mới học lớp 6 nha

sory

5 tháng 1 2018

Theo tính chất dãy tỉ số bằng nhau, ta có:

\(\frac{x}{y}=\frac{y}{z}=\frac{z}{x}=\frac{x+y+z}{y+z+x}=1\)

\(\Rightarrow x=y;y=z;z=x\Leftrightarrow x=y=z\)

Theo bài ra, ta có: \(x^{2017}-y^{2018}=0\)

\(\Rightarrow x^{2018}-x^{2017}=0\)

\(\Leftrightarrow x^{2017}.\left(x-1\right)=0\Leftrightarrow\hept{\begin{cases}x=0\left(bỏ\right)\\x=1\end{cases}}\)

Vậy x = y = z = 1

7 tháng 5 2018

Áp dụng BĐT Cauchy-Schwarz ta có:

\(\left(\dfrac{x^3}{y^2}+\dfrac{y^3}{z^2}+\dfrac{z^3}{x^2}\right)\left(x+y+z\right)\ge\left(\dfrac{x^2}{y}+\dfrac{y^2}{z}+\dfrac{z^2}{x}\right)^2\)

Cần chứng minh \(\dfrac{x^2}{y}+\dfrac{y^2}{z}+\dfrac{z^2}{x}\ge x+y+z\)

Dễ thấy;\(VT=\dfrac{x^2}{y}+\dfrac{y^2}{z}+\dfrac{z^2}{x}\ge\dfrac{\left(x+y+z\right)^2}{x+y+z}=x+y+z\)

BĐT được chứng minh

\("="\Leftrightarrow x=y=z\)

14 tháng 10 2021

\(x^3+y^3+z^3-3xyz=\left(x+y\right)^3+z^3-3xyz-3x^2y-3xy^2\)

\(=\left(x+y+z\right)\left[\left(x+y\right)^2-\left(x+y\right)z+z^2\right]-3xy\left(x+y+z\right)\)

\(=\left(x+y+z\right)\left[\left(x+y\right)^2-\left(x+y\right)z+z^2-3xy\right]\)

\(=0\)

\(\Rightarrow x^3+y^3+z^3=3xyz\)

3 tháng 1 2018

theo t/c dãy tỉ số bằng nhau ta có:

\(\dfrac{x}{y}=\dfrac{y}{z}=\dfrac{z}{x}=\dfrac{x+y+z}{y+z+x}=1\)

\(\Rightarrow x=y;y=z;z=x\Leftrightarrow x=y=z\)

theo bài ra ta có: \(x^{2017}-y^{2018}=0\)

\(\Rightarrow x^{2018}-x^{2017}=0\)

\(\Leftrightarrow x^{2017}\left(x-1\right)=0\)\(\Leftrightarrow\left\{{}\begin{matrix}x=0\left(loại\right)\\x=1\end{matrix}\right.\)

vậy x = y= z =1

31 tháng 12 2017

số 1 đấy

25 tháng 1 2022

giả sử cả 3 số xyz đều nhỏ hơn 1 

=>x+y+z<1+1+1=3

ta có x+y+z>\(\dfrac{1}{x}+\dfrac{1}{y}+\dfrac{1}{z}\)=\(\dfrac{xy+yz+xz}{xyz}\)\(\ge\)\(\dfrac{3\sqrt[3]{\left(abc\right)^2}}{abc}\) =\(\dfrac{3}{\sqrt[3]{abc}}=\dfrac{3}{\sqrt[3]{1}}=3\) vậy x+y+z >3

từ đó sẽ có ít nhất 1 trong 3 số lớn hơn 1